Question and Answers Forum

All Questions      Topic List

Algebra Questions

Previous in All Question      Next in All Question      

Previous in Algebra      Next in Algebra      

Question Number 92778 by unknown last updated on 09/May/20

Commented by prakash jain last updated on 09/May/20

⌊3x⌋−2−(⌊2x⌋−1)=2x−6  ⌊3x⌋−⌊2x⌋=2x−5   since LHS is an integer RHS  must be integer  ⇒x must be of form n or (n/2)  x is of form n  3n−2n=2n+7  n=5   x is of form (n/2)  ⌊((3n)/2)⌋−⌊2(n/2)⌋=((2n)/2)−5  ⌊((3n)/2)⌋−n=n−5  ⌊((3n)/2)⌋=2n−5  (A)  Now n must be of form 2k+1  other x will be of form n for which  we have already solved.  n=2k+1  ⌊((3(2k+1))/2)⌋=2(2k+1)−5  ⌊3k+1+(1/2)⌋=2(2k+1)−5  3k+1=4k−3  ⇒k=4 or n=9 or x=(9/2)  two solution  x=5 or x=(9/2)

$$\lfloor\mathrm{3}{x}\rfloor−\mathrm{2}−\left(\lfloor\mathrm{2}{x}\rfloor−\mathrm{1}\right)=\mathrm{2}{x}−\mathrm{6} \\ $$$$\lfloor\mathrm{3}{x}\rfloor−\lfloor\mathrm{2}{x}\rfloor=\mathrm{2}{x}−\mathrm{5}\: \\ $$$$\mathrm{since}\:\mathrm{LHS}\:\mathrm{is}\:\mathrm{an}\:\mathrm{integer}\:\mathrm{RHS} \\ $$$$\mathrm{must}\:\mathrm{be}\:\mathrm{integer} \\ $$$$\Rightarrow{x}\:\mathrm{must}\:\mathrm{be}\:\mathrm{of}\:\mathrm{form}\:{n}\:\mathrm{or}\:\frac{{n}}{\mathrm{2}} \\ $$$${x}\:\mathrm{is}\:\mathrm{of}\:\mathrm{form}\:{n} \\ $$$$\mathrm{3}{n}−\mathrm{2}{n}=\mathrm{2}{n}+\mathrm{7} \\ $$$${n}=\mathrm{5}\: \\ $$$${x}\:\mathrm{is}\:\mathrm{of}\:\mathrm{form}\:\frac{{n}}{\mathrm{2}} \\ $$$$\lfloor\frac{\mathrm{3}{n}}{\mathrm{2}}\rfloor−\lfloor\mathrm{2}\frac{{n}}{\mathrm{2}}\rfloor=\frac{\mathrm{2}{n}}{\mathrm{2}}−\mathrm{5} \\ $$$$\lfloor\frac{\mathrm{3}{n}}{\mathrm{2}}\rfloor−{n}={n}−\mathrm{5} \\ $$$$\lfloor\frac{\mathrm{3}{n}}{\mathrm{2}}\rfloor=\mathrm{2}{n}−\mathrm{5}\:\:\left({A}\right) \\ $$$$\mathrm{Now}\:{n}\:\mathrm{must}\:\mathrm{be}\:\mathrm{of}\:\mathrm{form}\:\mathrm{2}{k}+\mathrm{1} \\ $$$$\mathrm{other}\:{x}\:\mathrm{will}\:\mathrm{be}\:\mathrm{of}\:\mathrm{form}\:{n}\:\mathrm{for}\:\mathrm{which} \\ $$$$\mathrm{we}\:\mathrm{have}\:\mathrm{already}\:\mathrm{solved}. \\ $$$${n}=\mathrm{2}{k}+\mathrm{1} \\ $$$$\lfloor\frac{\mathrm{3}\left(\mathrm{2}{k}+\mathrm{1}\right)}{\mathrm{2}}\rfloor=\mathrm{2}\left(\mathrm{2}{k}+\mathrm{1}\right)−\mathrm{5} \\ $$$$\lfloor\mathrm{3}{k}+\mathrm{1}+\frac{\mathrm{1}}{\mathrm{2}}\rfloor=\mathrm{2}\left(\mathrm{2}{k}+\mathrm{1}\right)−\mathrm{5} \\ $$$$\mathrm{3}{k}+\mathrm{1}=\mathrm{4}{k}−\mathrm{3} \\ $$$$\Rightarrow{k}=\mathrm{4}\:\mathrm{or}\:{n}=\mathrm{9}\:\mathrm{or}\:{x}=\frac{\mathrm{9}}{\mathrm{2}} \\ $$$$\mathrm{two}\:\mathrm{solution} \\ $$$${x}=\mathrm{5}\:\mathrm{or}\:{x}=\frac{\mathrm{9}}{\mathrm{2}} \\ $$

Terms of Service

Privacy Policy

Contact: info@tinkutara.com